LSAT and Law School Admissions Forum

Get expert LSAT preparation and law school admissions advice from PowerScore Test Preparation.

 Administrator
PowerScore Staff
  • PowerScore Staff
  • Posts: 8916
  • Joined: Feb 02, 2011
|
#40707
Complete Question Explanation
(The complete setup for this game can be found here: lsat/viewtopic.php?t=3641)

The correct answer choice is (C)

In this question, you must select the answer choice that, when added to the existing rules, forces just a single solution to result. If you do not see an attractive answer immediately, the typical approach to such a question is to simply try the answer choices. But, never just start with answer choice (A) and work your way though until you find the right answer; instead, first try to analyze the answers in terms of the variables and their respective placement.

In Justify questions, the typical correct answer features a variable that is itself powerful (meaning that it affects other variables), and is then placed into a powerful spot on the diagram (meaning that it can more readily affect other variables from that particular position). Thus, you are looking for a strong variable in a power position.

As far as strong variables, randoms typically do not qualify, and thus in this game H is not likely to be the best variable to use as you attempt to solve this question. Interestingly, H is used in the first two answer choices, and thus, blindly trying answer choices (A) and (B) could easily cause you to waste some valuable time here at the end of the section.

The remaining three answer choices involve one of two variables—R or Y—which are each involved in sequences. Y is linked to just S (but S is also linked to Q), and R is linked to both J and G. Thus, the two variables are somewhat similar, although working with R is likely to be slightly easier (and thus faster). R is also a nutrition article, and therefore subject to the first rule, whereas Y is not linked directly to the first rule. Hence, of the two, R is the slightly preferable variable to examine first.

As you may remember from questions #18 and #19, placing articles into the fourth position results in a limited number of results, and so any answer choice that uses the fourth position would seem promising, especially since the fourth position is in the center of the diagram, and thus uniquely positioned to cause issues due to the first rule. That would mean that, preferably, examining a variable in the fourth position first would be beneficial (answer choice (C)), followed by the fifth position, which is the next closest to the center (answer choice (E)), and then finally the seventh position, which is furthest (answer choice (D)).

Adding up all of the information, the result is that answer choice (C) is the most likely correct answer based on a scan of its basic attributes. Let’s see what occurs when (C) is put into play:

If R is fourth, then due to the fourth rule J and G must be among the first three articles edited, with J :longline: G. But, J and G are both finance articles, and thus they cannot be consecutive per the first rule. Therefore, they must be first and third (and the second article must be a nutrition article since Y cannot be second):


PT68_Game_#4_#23_diagram 1.png
Because Q is not third, via the contrapositive of the second rule Q must be earlier than S (Q S). That can be connected to the sequence in the third rule, to produce a Q :longline: S :longline: Y chain. Because R, Q, and S are all nutrition articles, they cannot be consecutive, and thus Q :longline: S :longline: Y cannot be fifth, sixth, and seventh, respectively. Q must then be second. S must then be sixth and Y seventh, forcing H to be fifth:

PT68_Game_#4_#23_diagram 2.png
Accordingly, because R in fourth produces just a single solution, answer choice (C) is the correct answer.
You do not have the required permissions to view the files attached to this post.

Get the most out of your LSAT Prep Plus subscription.

Analyze and track your performance with our Testing and Analytics Package.